Refer to the attached image​

Refer To The Attached Image

Answers

Answer 1

Answer:     [tex]\boldsymbol{2(x-2)(5x+7)}[/tex]

========================================================

Work Shown:

[tex](4x^2-16)+3(2x-4)(x+1)\\\\4(x^2-4)+3*2(x-2)(x+1)\\\\4(x-2)(x+2)+3*2(x-2)(x+1) \ \text{ ... difference of squares rule}\\\\2*2(x-2)(x+2)+3*2(x-2)(x+1)\\\\2(x-2)(2(x+2)+3(x+1))\\\\2(x-2)(2x+4+3x+3)\\\\\boldsymbol{2(x-2)(5x+7)}\\\\[/tex]

The answer has been confirmed with WolframAlpha which is basically a calculator app, but does a ton more than any average calculator.

The CAS (computer algebra system) in the app GeoGebra is another tool I use all the time.

Yet another way to confirm the answer is to graph the original expression and the answer as separate functions. You should find they produce the exact same curve. One curve perfectly overlaps the other. This visually verifies the two expressions are the same thing, just written a different way of course.

Let me know if you have any questions about any step. The basic idea I did was to pull out the GCF and then combine like terms.


Related Questions

w and x are whole numbers
w>60
x<50
Work out the smallest possible value of x-z

Answers

Using the given information, the smallest possible value of x - z is 12

Evaluating an expression

From the question, we are to determine the smallest possible value of x - z

From the given information,

w and x are whole numbers

and

w > 60

x < 50

∴ w = 61, 62, 63, 63, 65, ...

x = 49, 48, 47, 46, 45, ...

Then,

The smallest possible value of x - z is

x - z = 61 - 49

x - z = 12

Hence, the smallest possible value of x - z is 12

Learn more on Evaluating an expression here: https://brainly.com/question/21245378

#SPJ1

Which of the following objects is considered to be one-dimensional because it ha length but no width or height?
A. Line segment
B. Point
C. Triangle
D.Cube

Answers

A line is a one-dimensional ger
straight -undefined figure whicl
thickness or depth. It extends
both directions.
A line segment is a part of a lin
points on both the sides.
It has only length therefore, it i:
dimensional.
Best possible solution would be

A. Line segment

5(b)
Joe has a bucket containing 1370cm³ of water measured to the nearest 10 cm³.
Joe Says
"If I tip my bucket of water in the cuboid container, it will never overflow"
Is Joe correct?
You must explain your answer

Answers

If the side length is greater than 11.11 cm then it will not overflow.

Otherwise, it will overflow.

If Joe tips the bucket of water in a cuboid container and the water is not overflowing then the cuboid container must be of volume greater than 1370 cm³.

We find the cube root of 1370 cm³.

[tex]\sqrt[3]{1370} \approx11.11[/tex]

Then the cuboid container should have a side of length greater than 11.11 cm.

Here the statement  "If I tip my bucket of water in the cuboid container, it will never overflow" is correct or wrong based on the information that the container has a side length lesser or greater than 11.11 cm.

If the side length is greater than 11.11 cm then it will not overflow.

Otherwise, it will overflow.

Learn more about volume here-

brainly.com/question/1578538

#SPJ10

9. Solve for x: Please explain

Answers

Answer:

since the triangle has a line on 2 sides, it means that it is equal. Also, every triangle has 180 degrees so the missing degree is 70

Step-by-step explanation:

Figured it out

Answer:

x = 70°

Step-by-step explanation:

Since 2 sides are marked as congruent then the triangle is isosceles with 2 base angles being congruent , both 55°

the sum of the 3 angles in the triangle = 180° , so

x + 55° + 55° = 180°

x + 110° = 180° ( subtract 110° from both sides )

x = 70°

can someone please help and explain its due in a littleee

Answers

Answer:

slope = 3

y-intercept: -4

explanation on how to graph given in step-by-step explanation

Step-by-step explanation:

y=3x-4 is given in slope-intercept form which is y=mx+b where m is the slope and b is the y-intercept (when x = 0)

So the first coordinate that is easy to graph is the y-intercept since it's just (0, b) or in this case (0, -4). So that's the first point you graph. Now from here you can use the slope to graph all the other points. The slope is 3, which essentially means every time x increases by 1, the y-value increases by 3. So from the y-intercept (0, -4) you're going to go one unit to the right and three units up which should give you the coordinates (1, -1). Then from that coordinate go one unit to the right and three units up which should get you (2, 2) then from that coordinate go one unit to the right and three units up which should get you to (3, 5). Now go back the y-intercept (0, -4) and instead of going to the right 1 and up 3 you're going to do the inverse and instead go left 1 and down 3. So from (0, -4) go to the left one unit and down 3 units which should get you to (-1, -7). And now with all these points plotted draw a line through all of them

Find the variance of the given
data rounded to the nearest
hundredth.
3, 4, 5, 7, 10, 12, 15
Σ (x - mean)²
n


=
=
[?]
Enter

Answers

Answer:

 17.14

Step-by-step explanation:

You have the data values {3, 4, 5, 7, 10, 12, 15}

To find the mean, [tex]\frac{3+4+5+7+10+12+15}{7} =8[/tex], yay a whole number, my favorite.

Then, [tex](3-8)^2+(4-8)^2+(5-8)^2+(7-8)^2+(10-8)^2+(12-8)^2+(15-8)^2=120[/tex].

Since n=7, σ^2≈[tex]17.14[/tex]

Use 8a+16c.
What is the value of the expression when a = –5 and c = –1? Enter your answer in the box.

Answers

8(-5) x 16(-1) =
-40 x -16 = -640
So the answer is -640

What is the nth term rule of the linear sequence below?
27, 25, 23, 21, 19, ...

Answers

Answer:

Step-by-step explanation:

Comment

This is an arithmetic series. It has the following givens.

a = 27     The first term

d = - 2       The difference between one term and the one behind it.

n = quite small

Tn = a + (n - 1)*d

tn = 27 - 2n  + 2

tn = 29 - 2n

Which of the following is equivalent to
(16 3/2)^1/2 ?

O 6
O 8
O 12
O 64

Answers

Step-by-step explanation:

[tex] = { ({16}^{ \frac{3}{2} } )}^{ \frac{1}{2} } [/tex]

[tex] = { ({( {2}^{4}) }^{ \frac{3}{2} } )}^{ \frac{1}{2} } [/tex]

[tex] = {2}^{4 \times \frac{3}{2} \times \frac{1}{2} } [/tex]

[tex] = {2}^{ \frac{12}{4} } [/tex]

[tex] = {2}^{3} [/tex]

[tex] = 2 \times 2 \times 2[/tex]

[tex] = 8[/tex]

The answer is B.

Last week, you went to the water park with friends on Monday, Wednesday, and Friday. If you stayed at the water park 3.5 hours each of those days, how many hours did you spend at the water park last week?
A. 15 hours
B. 9 hours
C. 5.5 hours
D. 10.5 hours

Answers

Answer: D. 10.5 hours

Step-by-step explanation:

3 days, 3.5 hours each

3.5 * 3 = 10.5 hours

Answer:

D. 10.5

Step-by-step explanation:

3.5 x 3 = 10.5

Find the simplified product. b-5/2b X b^2+3b/b-5

Answers

The simplified product of [tex]\frac{b- 5}{2b} X \frac{b^{2} + 3b }{b - 5}[/tex] is [tex]\frac{(b+3)}{2}[/tex]

How to simplify a product?

The products can be simplified as follows;

Multiplying fraction,

[tex]\frac{x}{y} . \frac{a}{b} = \frac{xa}{yb}[/tex]

Therefore,

[tex]\frac{b- 5}{2b} X \frac{b^{2} + 3b }{b - 5} = \frac{(b-5)(b^{2} + 3b )}{(2b)(b-5)}[/tex]

Hence,

Let's reduce the fraction by dividing

[tex]\frac{b- 5}{2b} X \frac{b^{2} + 3b }{b - 5} = \frac{(b-5)(b^{2} + 3b )}{(2b)(b-5)} = \frac{(b^{2}+3b )}{(2b)}[/tex]

Therefore,

[tex]\frac{b- 5}{2b} X \frac{b^{2} + 3b }{b - 5} = \frac{(b-5)(b^{2} + 3b )}{(2b)(b-5)} = \frac{(b^{2}+3b )}{(2b)} = \frac{(b(b+3))}{2b}[/tex]

Hence,

[tex]\frac{b- 5}{2b} X \frac{b^{2} + 3b }{b - 5} = \frac{(b-5)(b^{2} + 3b )}{(2b)(b-5)} = \frac{(b^{2}+3b )}{(2b)} = \frac{(b(b+3))}{2b} = \frac{b+3}{2}[/tex]

learn more on simplification here: https://brainly.com/question/3306438

#SPJ1

Answer:

A, b+3/2

Step-by-step explanation:

How many solutions does the following equation have?
7(y+3)=5y+8
A. No Solution
B. Exactly One Solution
C. Infinitely many solutions

Answers

B. Exactly one solution

7y +21 = 5y +8
7y +13=5y
13=-2y
-6.5=y

Answer:

B. Exactly One Solution

Step-by-step explanation:

Given:

[tex]7(y+3)=5y+8[/tex]

Expand the brackets:

[tex]\implies 7y + 21 = 5y + 8[/tex]

Subtract 21 from both sides:

[tex]\implies 7y + 21-21 = 5y + 8-21[/tex]

[tex]\implies 7y=5y-13[/tex]

Subtract 5y from both sides:

[tex]\implies 7y-5y=5y-13-5y[/tex]

[tex]\implies 2y=-13[/tex]

Divide both sides by 2:

[tex]\implies \dfrac{2y}{2}=\dfrac{-13}{2}[/tex]

[tex]\implies y=-\dfrac{13}{2}[/tex]

Therefore, the equation has exactly one solution.

what is 1,000,000,000 divided by -1,000

Answers

-1000000 that is the answer

hope this helps!

can someone help me?

Answers

Answer:

think of each T in the original equation as an empty box, like this:

P(   ) = 2(   )² + (   ) - 7

when it says p(some number), it means replace all Ts in the equation with that number.

P( 0 ) = 2( 0 )² + ( 0 ) - 7 = 0 + 0 - 7 = -7

do the same for the rest <:)

for the second part, it's the same concept: every T is replaced with what's inside the parentheses.

so P( -t ) = 2( -t )² + ( -t ) - 7 = 2t² - t - 7

and so on and so forth.

BC and DE are tangents to this circle. Work out the size of angle 0. Give reasons for your answer. A 52° B O D 27° E 0 C Not drawn accurately BC and DE are tangents to this circle . Work out the size of angle 0 . Give reasons for your answer . A 52 ° B O D 27 ° E 0 C Not drawn accurately​

Answers

Answer:

Θ = 39°

Step-by-step explanation:

OA = OB ( radii of the circle ) , then

Δ AOB is isosceles with base angles congruent , that is

∠ ABO = ∠ BAO = 52°

the exterior angle of a triangle is equal to the 2 opposite interior angles.

∠ DOB is an exterior angle of the triangle , so

∠ DOB = ∠ ABO + ∠ BAO = 52° + 52° = 114°

the angle between a tangent and the radius at the point of contact is 90°, then

∠ OBC = ∠ ODE = 90°

the sum of the 4 interior angles of quadrilateral OBCD = 360°

∠ OBC + ∠ DOB + ∠ ODC + ∠ BCD = 360°

90° + 114° + (90 + 27)° + Θ = 360°

90° + 114° + 117° + Θ = 360°

321° + Θ = 360° ( subtract 321° from both sides )

Θ = 39°

Which transformation would be a horizontal reflection and a dilation of scale factor ½ of the L shape?

Answers

The transformation which would be a horizontal reflection and a dilation of scale factor ½ of the L shape is as in the attached image.

What is horizontal reflection and dilation?

The term horizontal reflection with respect to transformation in mathematical context simple means that the shape is reflected over the vertical axis(y-axis as mirror). The dilation of a scale factor of ½ simply means that the shape reduces to half it's original size.

Hence, it follows that the attached image represents the transformation.

Read more on transformation;

https://brainly.com/question/4289712

#SPJ1

(PLEASE HELP!) A rectangular photograph measures 24x19 cm. It is mounted on a card of size such that there is a 2 cm border all around. For the purposes of this question take the length of the photograph to be 24 cm.

If one of the borders must be 2 cm in width, determine the width of a possible length and width so that the inner and outer rectangles are similar.

Answers

Answer: The card is 28 cm by 23 cm

Check out the diagram below

Explanation:

I added 4 cm to each dimension of the "24 x 19". Why 4 instead of 2? Because we're effectively adding two copies of "2 cm" to each of the original length and width. Along the horizontal component, we have the left and right boundaries. Along the vertical component, we have the top and bottom boundaries. The diagram hopefully clears up any confusion you may have.

The image of ΔABC after a reflection across Line E G is ΔA'B'C'.

2 triangles are shown. Line E G is the line of reflection. Line segment D D prime has a midpoint at point F. Line segment C C prime has a midpoint at point G. Points B and B prime share a point. Angle C G F is a right angle.
Which triangle must be a right triangle and why?

ΔA'B'C' is right because it is the image of ΔABC.
ΔADC is right because AA' intersects AC at A.
ΔBCC' is right because B lies of the line of reflection.
ΔBGC is right because Line E G is perpendicular-to CC'.

Answers

ΔDGC is right because Line E G is perpendicular-to CC', Option D is the correct answer.

What is Reflection ?

Reflection along a line is plotting a new object as a mirror image of the original object .

It is given that the there are two Triangles , ABC and A'B'C' as an image.

The figure is attached with the answer.

D is the mid point of the line joining BB'

As the line of reflection is always perpendicular to the object and the image.

And as it is given that C G F is a right angle.

Therefore triangle DGC is right angled triangle , (the option given in the question is incorrect ) ,  Line E G is perpendicular-to CC'.

Option D is the correct answer.

To know more about Reflection

https://brainly.com/question/2254222

#SPJ1

Answer:

D

Step-by-step explanation:

There are x boys in a class. If the
number of girls are 3x + 1, how many
boys and girls are in the class
altogether.

Answers

Answer:

4x+1

Step-by-step explanation:

3x+x+1=4x+1

Let f(x)=6/-2+2e^-0.3x. what is f(4)? enter your answer rounded to the nearest tenth

Answers

The value of the function f(x)=6/-2+2[tex]e^{-0.3x}[/tex] at x=4 after rounded to the nearest tenth is -2.4.

Given Function of x : f(x)=6/-2+2[tex]e^{-0.3x}[/tex]

The given function showing the relationship between x and f(x) is f(x)=6/-2+2[tex]e^{-0.3x}[/tex] where e is exponential and we have to find the value of f(4) by just putting the value of x=4 in the given function and round to the nearest tenth means replacing a number with an approximate value.

f(4)=6/-2+2[tex]e^{-0.3*4}[/tex]   ( The value of e is 2.7812)

f(4)=-3+2[tex]e^{-1.2}[/tex]                    

f(4)=-3+2*0.30119   (The value of [tex]e^{-1.2}[/tex] is 0.30119)          

f(4)=-3+0.60238

f(4)=-2.3972

Round -2.3972 to tenth is -2.4.

Hence the value of function of at x=4 is -2.4.

Learn more about function at https://brainly.com/question/10439235

#SPJ10

The value of f(4) is -4.3.

The function is a relation between two different sets X and set Y which are in one-one or many-one relation. Here set X is called as the domain and set Y is called as the codomain.

For example: f(x)=3x+9

To get the value of the function at any point of x, we have to put the number in the function to get its function value.

For example the value of f(x) in above function at x=a will be f(a)=3a+9

Similarly given function in the question

[tex]f(x)=\frac{6}{-2+2e^{-0.3x}}[/tex]

to get the value of f(x) at x=4 put the value of x in the function,

[tex]f(x)=\frac{6}{-2+2e^{-0.3*4}}[/tex]

⇒[tex]f(x)=\frac{6}{-2+2e^{-1.2}}[/tex]

⇒f(x)=-4.3

Therefore the value of f(4) is -4.3.

Learn more about Function

here: https://brainly.com/question/4025726

#SPJ10

+
Carla babysits and tutors to make extra money. She babysits for $5 an hour and tutors for $10 an hour.
She can only work at most 15 hours a week. She wants to go on a trip with her friend and needs to
make $95 dollars this week to cover the cost of the trip. Write a system of inequalities to represent this
situation.

Answers

The system of inequalities to represent this situation will be x + y ≤ 15 and 5x + 10y ≤ 95.

What is the linear system?

A linear system is one in which the parameter in the equation has a degree of one. It might have one, two, or even more variables.

Carla babysits and tutors to make extra money.

She babysits for $5 an hour and tutors for $10 an hour.

She can only work at most 15 hours a week.

Likewise, she wants to go on a trip with her friend and needs to make 95 dollars this week to cover the cost of the trip.

Then the system of inequalities to represent this situation will be

Let x be the number of babysits hours and y be the number of tutor's hours.

     x + y ≤ 15

5x + 10y ≤ 95

More about the linear system link is given below.

https://brainly.com/question/20379472

#SPJ1

80 pupils in a sports centre are surveyed. the pupils can only use the swimming pool and the gym. 32 pupils use the swimming pool. 43 pupils use the gym. 19 pupils use neither the swimming pool nor the gym. find the probability to select a pupil that uses the swimming pool but not the gym.

Answers

[tex]\frac{18}{80}[/tex] is the probability of pupil that uses the swimming pool but not the gym.

It is given that:

Total students are 80

32 pupils use the swimming pool

43 pupils use the gym

19 pupils use neither the swimming pool nor the gym

To know the pupils who use both add 32+43 = 75

75-61= 14 pupils use both

32-14 = 18 pupils use only the swimming pool

43-18= 29 pupils use only the gym

Hence, [tex]\frac{18}{80}[/tex] is the probability of pupil that uses the swimming pool but not the gym.

To know more about probability visit: https://brainly.com/question/13604758

#SPJ4

Matt is trying to measure the height of a tree using
trigonometry. He is having trouble because of the
terrain around the tree. The horizontal distance
from the tree to Matt's eyes is 120 feet. The angle
of depression from the horizontal is 30°. Matt's
angle of sight to the top of the tree is 23°. What is
the height of the tree? (Round to the nearest foot.)

Answers

The height of the tree will be 18.35 feet.

What is a right-angle triangle?

It's a form of a triangle with one 90-degree angle that follows Pythagoras' theorem and can be solved using the trigonometry function.

Matt is trying to measure the height of a tree using trigonometry.

He is having trouble because of the terrain around the tree.

The horizontal distance from the tree to Matt's eyes is 120 feet.

The angle of depression from the horizontal is 30°.

Matt's angle of sight to the top of the tree is 23°.

The diagram is given below.

Let x be the height of tree and AM be h.

The value of (h + x) will be

tan 30° = (x + h) / 120

   x + h = 69.288

Then the value of h will be

tan 23° = h / 120

        h = 50.94 feet

Then the height of the tree will be

⇒ h + x – h

⇒ 69.29 - 50.94.

⇒ 18.35 feet

More about the right-angle triangle link is given below.

https://brainly.com/question/3770177

#SPJ1

4 kg 2 dag when converted in g gives you?

Answers

Answer:

4020 grams

Step-by-step explanation:

4kg=4000g

2 dag= 20g

Total: 4000g + 20g = 4020g

Answer:

4020 grams

Step-by-step explanation:

1 kg = 1000 g
1 dag = 10 g
4 x1000= 4000
2 x 10 = 20
4000 + 20

Write and solve a proportion to complete the statement. Round to the nearest hundreth if necessary.
1. 6km ≈ ?mi
2. 2.5 L≈ ?gal
3. 90lb ≈ ?kg
pls help

Answers

The conversion of the following units are done below.

6 km = 3.72 miles2.5 L = 0.65 gallons90 lb = 40.5 kg

What is conversion?

Conversion means to convert the same thing into different units.

We know the following unit's conversion.

1 km = 0.62 miles

6 km = 3.72 miles

    1 L = 0.26 gallons

2.5 L = 0.65 gallons

  1 lb = 0.45 kg

90 lb = 40.5 kg

More about the conversion link is given below.

https://brainly.com/question/9414705

#SPJ1

Identify a pair of overlapping congruent triangles in the
diagram. Then use the given information to write a proof
to show that the triangles are congruent.
Proof
Given: AC = BC, LA = LB

Answers

Answer:

overlapping congruent triangles:

ΔACE ≅ ΔBCD

Proof:

given: AC ≅ BC, angle A ≅ angle B

angle ACB ≅ angle BCA by the reflexive property

ΔACE ≅ ΔBCD by the ASA triangle congruence property

f(x)=x+2 g(x)=x-4
(f g)(x) =
O2x-2
O²-8
02-2x-8
Ọ x2+2x-8
Solve for (f g)(x)

Answers

Step-by-step explanation:

please mark me as brainlest

The new function (fog)x will be x - 2.

What is the function?

A relationship between a group of inputs and one output is referred to as a function. In plain English, a function is an association between inputs in which each input is connected to precisely one output. A domain, codomain, or range exists for every function. Typically, f(x), where x is the input, is used to represent a function.

To find (f g)(x), we need to evaluate f(g(x)).

First, we need to find g(x):

g(x) = x - 4

Next, we substitute g(x) into f(x):

f(g(x)) = f(x - 4)

Now we can simplify:

f(x - 4) = (x - 4) + 2

f(g(x)) = x - 2

Therefore, (f g)(x) = x - 2.

Learn more about function here:

https://brainly.com/question/29633660

#SPJ7

help grade 5 math please only answer if you know and if it’s right ill mark brainliest and please fast extra points who explain it

Answers

Answer:

[tex]\frac{42}{8}[/tex]

Step-by-step explanation:

7/8 <> 0.875

0.875 * 6 = 5.25

5.25 <> 42/8

(<> is used to show decimal to fraction and vice versa [the other way around])

An indoor running track is 200 meters in length. During a 3,000-meter race, runners must complete 15 laps of the track. An electronic timing device records the amount of time it takes each runner to complete a lap for every lap in the race. These are called lap times. The histograms below display the lap times for both Stefano and Alex, runners in the 3,000-meter race.

2 histograms. A histogram titled Alex apostrophe s 3,000 meter race lap times has lap times (seconds) on the x-axis, and frequency on the y-axis. 35 to 36, 1; 36 to 37, 5; 37 to 38, 4; 38 to 39, 4; 39 to 40, 1. A histogram titled Stefano apostrophe s 3,000 meter race lap times has lap times (seconds) on the x-axis, and frequency on the y-axis. 32 to 33, 1; 34 to 35, 1; 36 to 37, 1; 37 to 38, 4; 38 to 39, 5; 39 to 40, 1; 40 to 41, 2.

Using the histograms, which of the following is a correct comparison?

Neither runner had a lap time between 35 and 36 seconds.
Alex’s lap times show less variability than Stefano’s lap times.
The median lap time for both runners is in the 38–39 interval.
Both runners’ highest number of lap times in the 37–38 interval.

Answers

The true statement based on the histogram is

There were no lap times between 35 and 36 seconds.

The correct option is (A).

What is Histogram?

A histogram is a graphical representation that organizes a group of data points into user-specified ranges.

Given:

indoor running track is 200 meters in length.

Also, a 3,000-meter race, runners must complete 15 laps of the track.

As, there no data for interval 35-36.

So, there were no lap times between 35 and 36 seconds.

Hence, The interval from 37 to 38 seconds saw the most lap times.

Learn more about histogram here:

brainly.com/question/16819077

#SPJ1

Answer:

B. Alex's lap times show less variability than Stefano's lap times.

Step-by-step explanation:

A can't be right because Alex has a lap time between 35 and 36 seconds.

C can't be right because the median lap time for Alex isn't 38-39.

D can't be right because both runners have a higher frequency on lap times other than 37-38.

so it has to be B.

To eliminate the × terms and solve for y in the fewest steps by which constants should be equations be multiplied by before adding the equations together

Answers

To eliminate the y terms and solve for x in the fewest steps, you can multiply the equations by appropriate constants so that the coefficients of the y terms in Both equations have the same magnitude but opposing signs. So to eliminate the y terms and solve for x in the fewest steps, you should multiply the first equation by 2 and the second equation by 3x

In this instance, you can do the following steps:

Start with the original equations:

Equation 1: 2x + 3y = 8

Equation 2: 3x - 2y = 4

To eliminate the y terms, multiply Equation 1 by 2 and Equation 2 by 3:

Multiply Equation 1 by 2:

2(2x + 3y) = 2(8)

4x + 6y = 16

Multiply Equation 2 by 3:

3(3x - 2y) = 3(4)

9x - 6y = 12

Now, add the two modified equations together to eliminate the y terms:

(4x + 6y) + (9x - 6y) = 16 + 12

4x + 9x + 6y - 6y = 28

13x = 28

Finally, solve for x by dividing both sides of the equation by 13:

x = 28/13

Therefore, to eliminate the y terms and solve for x in the fewest steps, you should multiply the first equation by 2 and the second equation by 3x

For more questions on: constants

https://brainly.com/question/27983400

#SPJ8        

The probable question may be:

To eliminate the y terms and solve for x in the fewest steps, by which constants should the equations be multiplied by before adding the equations together? The first equation should be multiplied by 2 and the second equation by 3. x.

Other Questions
A solution is 0.225M Na2SO4. What is the molarity of the sodium ion in solution In which identity category do you think you could benefit from additional education for your personal and professional growth? Sort the following identity categories in order of your knowledge. Place the identity in which you could benefit from the most learning/growth at the top, with the rest of the identity categories following in descending order. When you are done with this question, the identity at the top of the list is the one you know least about, and the identity at the bottom of the list is the one you feel you know the most about. Identify the reaction type for each unrelated, radical reaction as either initiation, propagation, or termination. Reaction A. A chlorine chlorine bond splits into two chlorine radicals. What type of reaction is reaction A A parallelogram ABCD is shown with DC equal to 8 feet and the perpendicular distance between AB and DC equal to 3 over 4 foot. Part A: What is the area of the parallelogram? Show your work. (5 points) Part B: How can you decompose this parallelogram into two triangles? If this parallelogram was decomposed into two triangles, what would be the area of each triangle? (5 points) Find the following, giving your answersto 1 decimal place.10 cm/6 cm8 cma) The volume of the cone.b) The curved surface area of the cone. Pls answer these for me! Which criteria should Miralee use for her selection?Check all that apply. if you are to write a novel and you want to pattern it from someone else work what do you call this Compltez avec le pronom rflchi correct (me [m], te [t], etc.).Est-ce que tu___maquilles tous les jours?Elle ___ couche normalement 23 h? -356= -3506-350-6= 350 - 60 = NEED THIS ASAP! Britain, with an army to enforce her tyranny, has declared that she has a right (not only to TAX) but to BIND us in ALL CASES WHATSOEVER, and if being bound in that manner, is not slavery, then is there not such a thing as slavery upon earth. . . .. . . Neither have I so much of the infidel in me, as to suppose that He has relinquished the government of the world, and given us up to the care of devils; and as I do not, I cannot see on what grounds the king of Britain can look up to heaven for help against us: a common murderer, a highwayman, or a house-breaker, has as good a pretence as he.The words slavery, devils, and murderer evoke a sense ofA.anger and injustice.B. fear and anxiety.C. confusion and chaos.D. guilt and shame. Tatenda is making tea andtoast. He uses an 800Wtoaster for 2.5 minutes anda 2kW kettle for 5 minutes.What is the total cost forthe energy used ifelectricity costs 10p perunit? Recently you witnessed a disturbance at a railway station when many people were injured. You are asked by the local police to write an account of what you saw. Write your eyewitness report account. ________ development involves learning, attention, memory, language, thinking, reasoning, and creativity. Read Shakespeare's "Sonnet 100."Where art thou, Muse, that thou forget'st so longTo speak of that which gives thee all thy might?Spend'st thou thy fury on some worthless song,Darkening thy power to lend base subjects light?Return, forgetful Muse, and straight redeemIn gentle numbers time so idly spent;Sing to the ear that doth thy lays esteemAnd gives thy pen both skill and argument.Rise, resty Muse, my love's sweet face survey,If Time have any wrinkle graven there;If any, be a satire to decay,And make Time's spoils despised every where.Give my love fame faster than Time wastes life;So thou prevent'st his scythe and crooked knife.What is the central idea of the third quatrain?If you see my love aging, work against it.Do not be lazy; we are all running out of time.Make fun of time, and turn people against it.Write something to ridicule my love's wrinkles. can someone confirm why it's not? im *pretty* sure it's d? Whether supplier-seller relationships in an industry represent a strong or weak source of competitive pressure is a function of a. whether the profits of suppliers are relatively high or low. b. the average number of suppliers that each seller/industry member purchases from. c. how aggressively rival industry members are trying to differentiate their products. d. whether demand for supplier products is high and they are in short supply. e. whether the prices of the items being furnished by the suppliers are rising or falling. A system of equations has 1 solution. If 4x - y = 5 is one of the equations, which could be the other equation?A.y=-4x+5B. Y=4x -5C.2y=8x-10D.-2y=-8x-10 Describe the translation from f ( x ) = I x I... to f ( x ) = I x - 1 I + 11. what position did a British politician play?